Non-empty perfect set in R with no rational number

In summary: My bad.In summary, the conversation discusses the question of whether there exists a non-empty perfect set that contains no rational numbers. The initial answer is no, but the professor provides a counterexample. The conversation then delves into different constructions for such a set, with one eventually being proven flawed. Ultimately, it is determined that the intersection of an interval with the complement of the rationals is not a perfect set due to a lack of closure.
  • #1
PingPong
62
0

Homework Statement


Is there a non-empty perfect set that contains no rational number?


Homework Equations


None


The Attempt at a Solution


I thought the answer was no, but my professor said that there is. My reasoning is as follows (please let me know if I'm wrong here):

If p is an irrational limit point of a perfect set P, then every open ball B(p;r) around the point such that B(p;r) that contains another point in P. But this ball contains rational numbers, so a rational number q is in B(p;r). Thus a ball of the same radius around q contains the point p, which is in P. So q is a limit point of P (because r was arbitrary). Since P must be closed, it contains all of its limit points, so q is in P.

Where'd I mess up? Thanks in advance!
 
Physics news on Phys.org
  • #2
PingPong said:
So q is a limit point of P (because r was arbitrary).

This doesn't follow - there may be different q for different r.

I agree with your professor. To get the example I have in mind, start by trying to construct an open set containing the rationals whose complement has positive measure.
 
  • #3
Ok, I can see where I messed up, but now I haven't got a clue about where to start. We haven't gone over anything with measure - the only tools I've got are those presented in Rudin's Principles of Mathematical Analysis, chapters 1-2.

Thanks again!
 
  • #4
You can construct a "fat Cantor set" in [0,1] that misses all the rationals (or any other countable set of your choosing). These things I believe still have the property of being perfect.
 
  • #5
Instead of creating a new thread, I decided to just write in here.

If we define a set X={p in (0, 1), p in R} and let E = X \ Q.

Now, E non-empty(since Q is countable and X is not) and bounded, furthermore, no point of Q is in E. Now what remains to be shown is that if p in E, then p is a limit point of E. Clearly, every neighborhood of p contains another irrational(this point can made rigorous, I suppose) and since E does not contain any rational, the neighborhood doesn't either.
So E is the required set.

Is there a flaw in the (naive) argument somewhere? The answers I've seen so far proceed with a construction of E by removing neigborhoods around enumaration of rational, etc.
 
  • #6
C'mon, guys. There must be something wrong with this naive construction, otherwise it would be used as an example(on planetmath for instance).
 
  • #7
There are several flaws in your argument a_Vatar. If the set E is, as you described, the set of all real numbers p in the interval (0, 1) such that p is not rational, then clearly 0 is a limit point of E. To see this, note that sqrt(2)/2 is contained in E, as is sqrt(2)/(2^n), for any positive integer n. By the archimedean property of R, there exists n such that sqrt(2)/(2^n)< r for any positive real number r. This shows therefore that there is a point of E in every neighborhood of 0, and so 0 is a limit point of E. Since 0 is not contained in E, E is not closed, and hence not perfect.

A more general argument can be made, in fact, which shows that for any interval I of R (open or closed), the intersection of I with the complement of the set Q of all rationals is not perfect. In order to prove this, suppose to the contrary that there exists such a set, say P, and suppose q is a rational number such that q is contained in I and q is not an endpoint of I (we are guaranteed such a q by the density of Q in R). Since P is perfect, q is not a limit point of P, hence there exists a neighborhood of q which is a subset of I and which contains no point of P. It follows that there exists a subinterval I* of I which contains only rational numbers. However, I* is clearly countable, which contradicts the fact that every interval of real numbers is uncountable.
 
  • #8
I fail to see why [itex][e,\pi]\bigcap \mathbb{Q}^c[/itex] is not a perfect set...
 
  • #9
Oh gosh nevermind, it isn't closed.
 

1. What is a non-empty perfect set in R?

A non-empty perfect set in R is a set of real numbers that is both closed and uncountable. This means that the set contains all of its limit points and cannot be put into a one-to-one correspondence with the set of natural numbers.

2. How is a perfect set different from a closed set?

A perfect set is a subset of a closed set. While a closed set may have a finite number of limit points, a perfect set contains all of its limit points and has an infinite number of elements.

3. What is the significance of a perfect set having no rational numbers?

A perfect set with no rational numbers is an example of a dense-in-itself set, meaning that every point in the set is a limit point. This property makes perfect sets useful in the study of topology and real analysis.

4. How do you prove that a set is a non-empty perfect set in R with no rational numbers?

To prove that a set is a non-empty perfect set in R with no rational numbers, one must show that the set is closed, uncountable, and contains no rational numbers. This can be done through various methods, such as using the Cantor diagonalization argument.

5. Are there any real-world applications of non-empty perfect sets in R with no rational numbers?

Perfect sets have various applications in mathematics and physics, including the study of fractals and chaotic systems. Additionally, perfect sets are used in the construction of the famous Cantor set, which has applications in computer graphics and data compression.

Similar threads

  • Calculus and Beyond Homework Help
Replies
2
Views
2K
  • Calculus and Beyond Homework Help
Replies
8
Views
1K
  • Calculus and Beyond Homework Help
Replies
8
Views
1K
  • Calculus and Beyond Homework Help
Replies
5
Views
1K
  • Calculus and Beyond Homework Help
Replies
7
Views
1K
  • Calculus and Beyond Homework Help
Replies
3
Views
1K
  • Calculus and Beyond Homework Help
Replies
7
Views
1K
Replies
9
Views
324
  • Calculus and Beyond Homework Help
Replies
1
Views
5K
  • Science and Math Textbooks
Replies
10
Views
2K
Back
Top